Solving an Initial Value Problem for x(t=ln2): Step-by-Step Solution

Click For Summary
The discussion focuses on solving the initial value problem for the differential equation dx/dt = x(2-x) with the condition x(0) = 1, specifically for x(t=ln2). The initial attempts involved integrating and manipulating the equation, but the user initially arrived at an incorrect solution of x=2/11. Participants pointed out errors related to the handling of absolute values and constants during integration. After clarifying the correct approach and substituting values, the user successfully found x=8/5, which matched one of the possible answers. The collaborative effort emphasized the importance of careful algebraic manipulation in solving differential equations.
Drakkith
Mentor
Messages
23,198
Reaction score
7,679

Homework Statement


Solve the initial value problem:
##\frac{dx}{dt} = x(2-x)##, ##x(0) = 1##
for ##x(t=ln2)##.

Homework Equations

The Attempt at a Solution



I moved the right side to the left and multiplied both sides by dt to get:
##\frac{dx}{x(2-x)} = dt##

Integrating gave me:
##\frac{ln|x|}{2} - \frac{ln|x-2|}{2} = t+C##

Then:
##ln|x| - ln|x-2| = 2t + 2C##
##ln|\frac{x}{x-2}| = 2t + 2C##
##\frac{x}{x-2} = e^{2t}e^{2c}##
##\frac{1}{1-\frac{2}{x}} = Ke^{2t}##

Manipulating for a while, I end up with:
##x=\frac{2}{Ke^{2t}}##

Since ##x(0) = 1##, I set the left side to 1 and solve for K, winding up with ##k=3##.
However, when trying to solve for ##x(ln2)## I end up with ##\frac{2}{11}##, which isn't one of my possible answers.

Does my process look even remotely correct?
 
Physics news on Phys.org
Drakkith said:

Homework Statement


Solve the initial value problem:
##\frac{dx}{dt} = x(2-x)##, ##x(0) = 1##
for ##x(t=ln2)##.

Homework Equations

The Attempt at a Solution



I moved the right side to the left and multiplied both sides by dt to get:
##\frac{dx}{x(2-x)} = dt##

Integrating gave me:
##\frac{ln|x|}{2} - \frac{ln|x-2|}{2} = t+C##

Then:
##ln|x| - ln|x-2| = 2t + 2C##
##ln|\frac{x}{x-2}| = 2t + 2C##
##\frac{x}{x-2} = e^{2t}e^{2c}##
##\frac{1}{1-\frac{2}{x}} = Ke^{2t}##
So you could say ##\frac x {x-2} = Ke^{2t}## Put ##t=0,~x=1## there to get ##\frac x {x-2} = -e^{2t}##. Now put ##t=\ln 2## in that, then solve for ##x##. See if that fixes it.
 
  • Like
Likes Drakkith
Drakkith said:

Homework Statement



****************.

##ln|x| - ln|x-2| = 2t + 2C##
##ln|\frac{x}{x-2}| = 2t + 2C##
##\frac{x}{x-2} = e^{2t}e^{2c}##
##\frac{1}{1-\frac{2}{x}} = Ke^{2t}##

For ##t## near 0 you have ##x## near 1, and so ##x/(x-2) = e^{2C} e^{2t}## is impossible unless you let ##C## be a complex number. However, ##\ln(|x|/|x-2|) = \ln(x/(2-x))##, and so having ##x/(2-x) = e^{2C} e^{2t}## is OK. Of course, when you re-wrote ##e^{2C}## as ##K## and then forgot the origin of ##K##, you were then able to have a valid formula!
 
  • Like
Likes Drakkith
Drakkith said:
##ln|\frac{x}{x-2}| = 2t + 2C##
##\frac{x}{x-2} = e^{2t}e^{2c}##
##\frac{1}{1-\frac{2}{x}} = Ke^{2t}##

Ray Vickson said:
For ##t## near 0 you have ##x## near 1, and so ##x/(x-2) = e^{2C} e^{2t}## is impossible unless you let ##C## be a complex number. However, ##\ln(|x|/|x-2|) = \ln(x/(2-x))##, and so having ##x/(2-x) = e^{2C} e^{2t}## is OK. Of course, when you re-wrote ##e^{2C}## as ##K## and then forgot the origin of ##K##, you were then able to have a valid formula!

@Drakkith: Ray is quite correct and I didn't notice you had been sloppy with the absolute value signs. A better way for you to have written it would be like this:$$
ln\left | \frac{x}{x-2}\right | = 2t + 2C$$ $$
\left|\frac{x}{x-2}\right | =e^{2t + 2C}=e^{2C}e^{2t}$$ $$
\frac{x}{x-2} =\pm e^{2C}e^{2t} = Ke^{2t}$$
 
  • Like
Likes Drakkith
LCKurtz said:
So you could say ##\frac x {x-2} = Ke^{2t}## Put ##t=0,~x=1## there to get ##\frac x {x-2} = -e^{2t}##. Now put ##t=\ln 2## in that, then solve for ##x##. See if that fixes it.

Yes, that seemed to work. I got x=8/5, which is one of the possible answers. Thanks guys!
 
Question: A clock's minute hand has length 4 and its hour hand has length 3. What is the distance between the tips at the moment when it is increasing most rapidly?(Putnam Exam Question) Answer: Making assumption that both the hands moves at constant angular velocities, the answer is ## \sqrt{7} .## But don't you think this assumption is somewhat doubtful and wrong?

Similar threads

  • · Replies 11 ·
Replies
11
Views
3K
Replies
5
Views
2K
Replies
5
Views
2K
  • · Replies 15 ·
Replies
15
Views
2K
  • · Replies 1 ·
Replies
1
Views
1K
Replies
4
Views
2K
  • · Replies 9 ·
Replies
9
Views
1K
  • · Replies 5 ·
Replies
5
Views
1K
  • · Replies 5 ·
Replies
5
Views
1K
  • · Replies 6 ·
Replies
6
Views
2K